Surface Integral for Curl of Vector Field V over a Parametrized Triangle

Click For Summary
The discussion focuses on the calculation of a surface integral for the curl of a vector field V over a parametrized triangle. The user is struggling with the integral, having obtained an incorrect result of -53/3, while the line integral around the triangle yields -35/6, which should match according to Stokes' theorem. A suggestion is made to change the parameterization to a more intuitive one using x and y, simplifying the calculation of dS. This adjustment would lead to correct limits over the triangular region in the xy-plane. The conversation emphasizes the importance of proper parameterization in surface integrals for accurate results.
theneedtoknow
Messages
169
Reaction score
0
I asked for some help on how to do surface integrals, and this is what I understood from that applied to a question, which i am getting the wrong answer to, so can someone please let me know which part of this I am doing wrong?

I need to find the integral of the curl of V, over the triangle with vertices (1,0,0), (0,3,0), (0,0,2), where V = (zx^2, 2yz, 0)

So the curl of V is (-2y, x^2, 0)
the surface parametrized is x = 1-s-t, y=3s, z=2t
so I need to dot the curl of V with d(1-s-t,3s,2t)/ds cross d(1-s-t,3s,2t)/dt = (-1,3,0)cross(-1,0,2) = (6, 2, 3)

So I have to integrate (-2(3s) , (1-t-s)^2, 0) dot (6, 2, 3) dsdt from s=0 to s=1 and from z=0 to z = 1

This is the double integral of (-36s + 2 -4t -4s + 4ts + 2t^2 + 2s^2) = (2-40s-4t+4ts+2t^2+2s^2) for s,t going from 0 to 1
but when i do this integral, i get -53/3

I have done the integral of V over the closed path around the triangle, and got -35/6
By stokes theorem these should be equal, and I am pretty sure my path integrals are correct at -35/6, so I must be doing something wrong with the surface integral. Please help!
 
Physics news on Phys.org
I haven't checked all your arithmetic, but almost certainly your limits for s and t are wrong. Why use such a non-intuitive parameterization? Your surface is just a plane:

\frac x 1 + \frac y 3 + \frac z 2 = 1

or

z = 2 - 2x -\frac 2 3 y

so it would be very natural to use, for example, x and y as parameters:

\vec R(x,y) = \langle x, y, 2 - 2x -\frac 2 3 y\rangle

Use that to calculate your dS in terms of dx and dy and your limits will be over the triangular region in the xy plane. The inner upper limit won't be constant.
 
thank you very much :)
 
Question: A clock's minute hand has length 4 and its hour hand has length 3. What is the distance between the tips at the moment when it is increasing most rapidly?(Putnam Exam Question) Answer: Making assumption that both the hands moves at constant angular velocities, the answer is ## \sqrt{7} .## But don't you think this assumption is somewhat doubtful and wrong?

Similar threads

  • · Replies 2 ·
Replies
2
Views
1K
  • · Replies 7 ·
Replies
7
Views
2K
  • · Replies 4 ·
Replies
4
Views
2K
  • · Replies 9 ·
Replies
9
Views
1K
Replies
5
Views
2K
  • · Replies 3 ·
Replies
3
Views
2K
Replies
6
Views
3K
  • · Replies 3 ·
Replies
3
Views
2K
  • · Replies 1 ·
Replies
1
Views
2K
  • · Replies 1 ·
Replies
1
Views
2K